$x^4+x^3+x^2+x+1$ irreducible over $mathbb F_7$

The name of the pictureThe name of the pictureThe name of the pictureClash Royale CLAN TAG#URR8PPP











up vote
7
down vote

favorite
5












This question came from the answer here.



The answer claims that $x^4+x^3+x^2+x+1$ is irreducible over $mathbb F_7$. I can check that it has no roots in the field, but why can't it be written as a product of two quadratics? I tried the method of undetermined coefficients: $$x^4+x^3+x^2+x+1=(x^2+ax+b)(x^2+cx+d)=\ x^4+(a+c)x^3+(ac+b+d)x^2+(bc+ad)x+bd$$ so $$a+c=1\ac+b+d=1\bc+ad=1\bd=1$$ but I wasn't able to solve this. Maybe there is a way to see irreducibility by only using general finite fields theory?







share|cite|improve this question



















  • Worst case scenario, check the different choices for $b$ and $a$. $c$ and $d$ are dictated at that point. I'd like to see a slicker way.
    – Randall
    Aug 6 at 19:38










  • this question may help.
    – lulu
    Aug 6 at 19:43














up vote
7
down vote

favorite
5












This question came from the answer here.



The answer claims that $x^4+x^3+x^2+x+1$ is irreducible over $mathbb F_7$. I can check that it has no roots in the field, but why can't it be written as a product of two quadratics? I tried the method of undetermined coefficients: $$x^4+x^3+x^2+x+1=(x^2+ax+b)(x^2+cx+d)=\ x^4+(a+c)x^3+(ac+b+d)x^2+(bc+ad)x+bd$$ so $$a+c=1\ac+b+d=1\bc+ad=1\bd=1$$ but I wasn't able to solve this. Maybe there is a way to see irreducibility by only using general finite fields theory?







share|cite|improve this question



















  • Worst case scenario, check the different choices for $b$ and $a$. $c$ and $d$ are dictated at that point. I'd like to see a slicker way.
    – Randall
    Aug 6 at 19:38










  • this question may help.
    – lulu
    Aug 6 at 19:43












up vote
7
down vote

favorite
5









up vote
7
down vote

favorite
5






5





This question came from the answer here.



The answer claims that $x^4+x^3+x^2+x+1$ is irreducible over $mathbb F_7$. I can check that it has no roots in the field, but why can't it be written as a product of two quadratics? I tried the method of undetermined coefficients: $$x^4+x^3+x^2+x+1=(x^2+ax+b)(x^2+cx+d)=\ x^4+(a+c)x^3+(ac+b+d)x^2+(bc+ad)x+bd$$ so $$a+c=1\ac+b+d=1\bc+ad=1\bd=1$$ but I wasn't able to solve this. Maybe there is a way to see irreducibility by only using general finite fields theory?







share|cite|improve this question











This question came from the answer here.



The answer claims that $x^4+x^3+x^2+x+1$ is irreducible over $mathbb F_7$. I can check that it has no roots in the field, but why can't it be written as a product of two quadratics? I tried the method of undetermined coefficients: $$x^4+x^3+x^2+x+1=(x^2+ax+b)(x^2+cx+d)=\ x^4+(a+c)x^3+(ac+b+d)x^2+(bc+ad)x+bd$$ so $$a+c=1\ac+b+d=1\bc+ad=1\bd=1$$ but I wasn't able to solve this. Maybe there is a way to see irreducibility by only using general finite fields theory?









share|cite|improve this question










share|cite|improve this question




share|cite|improve this question









asked Aug 6 at 19:31









user437309

556212




556212











  • Worst case scenario, check the different choices for $b$ and $a$. $c$ and $d$ are dictated at that point. I'd like to see a slicker way.
    – Randall
    Aug 6 at 19:38










  • this question may help.
    – lulu
    Aug 6 at 19:43
















  • Worst case scenario, check the different choices for $b$ and $a$. $c$ and $d$ are dictated at that point. I'd like to see a slicker way.
    – Randall
    Aug 6 at 19:38










  • this question may help.
    – lulu
    Aug 6 at 19:43















Worst case scenario, check the different choices for $b$ and $a$. $c$ and $d$ are dictated at that point. I'd like to see a slicker way.
– Randall
Aug 6 at 19:38




Worst case scenario, check the different choices for $b$ and $a$. $c$ and $d$ are dictated at that point. I'd like to see a slicker way.
– Randall
Aug 6 at 19:38












this question may help.
– lulu
Aug 6 at 19:43




this question may help.
– lulu
Aug 6 at 19:43










4 Answers
4






active

oldest

votes

















up vote
11
down vote



accepted










Any element in any field satisfying $x^4+x^3+x^2+x+1=0$ also satisfies $x^5=1$. In other words, it has order $5$ in the multiplicative group of that field.



The field with $p^k$ elements has a multiplicative group with $p^k-1$ elements, so it has elements of order $5$ if and only if $5$ divides $p^k-1$. This is not the case for $p=7$ and $k=1,2,3$, but it is the case for $p=7,k=4$.



Therefore, the smallest field containing $mathbbF_7$ and roots of $x^4+x^3+x^2+x+1$ has $7^4$ elements. If that polynomial were reducible, there would be a smaller splitting field with such roots, but there isn’t.






share|cite|improve this answer





















  • Why would there be a smaller splitting field (than that with $7^4$ elements) with the roots of $x^4+x^3+x^2+x+1$ if that polynomial were reducible?
    – user437309
    Aug 6 at 20:00










  • Take a factor of that polynomial and consider the splitting field of that factor.
    – Alon Amit
    Aug 6 at 20:01






  • 1




    I have lost count of the questions I have settled here using this argument. Welcome to the club :-)
    – Jyrki Lahtonen
    Aug 6 at 21:12










  • @JyrkiLahtonen proud to be a member!
    – Alon Amit
    Aug 6 at 22:08






  • 1




    @user437309 yes, that’s right.
    – Alon Amit
    Aug 7 at 19:45

















up vote
3
down vote













If you want to continue your approach in case you cannot come up with some clever argument like that in Alon Amit's answer, then you can do it as follows. First, note that $c=1-a$ and $d=b^-1$. Thus, the equation $ac+b+d=1$ becomes
$$a(1-a)+b+b^-1=1,.tag*$$
The equation $bc+ad=1$ is now
$$b(1-a)+ab^-1=1,.$$
Thus,
$$a^-1big(1-b(1-a)big)=b^-1=1-b-a(1-a),.$$
That is,
$$1-b(1-a)=a-ab-a^2(1-a),.$$
Consequently,
$$(2a-1)b=-1+a-a^2+a^3=(a-1)(a^2+1),.tag#$$
Thus, $anotin0,1,4$ for the equation above to be possible (noting that $aneq 0$ and $bneq 0$). You are left with four choices of $a$, namely, $ain2,3,5,6$.



If $a=2$, then, using (#), we get $b=3^-1cdot 5=5cdot 5=4$, so $b^-1=2$, but then $$a(1-a)+b+b^-1=2cdot(-1)+4+2=4neq 1,.$$
If $a=3$, then we have by (#) that $b=5^-1cdot20=-3=4$, whence
$$a(1-a)+b+b^-1=3cdot(-2)+4+2=0neq 1,.$$
If $a=5$, then $b=3^-1cdot (-1)=-5=2$, so $b^-1=4$, but then
$$a(1-a)+b+b^-1=5cdot(-4)+2+4=0neq 1,.$$
Finally, if $a=6$, then $b=4^-1cdot3=2cdot 3=-1$, whence $b^-1=-1$, and
$$a(1-a)+b+b^-1=6cdot(-5)+(-1)+(-1)=-4neq 1,.$$
Thus, (*) cannot be satisfied.






share|cite|improve this answer




























    up vote
    1
    down vote













    Yet an other solution. It is based on the fact that
    $$ f = x^4+x^3+x^2+x+1inBbb F_7[x] $$
    is reciprocal, and tries to use the idea in the OP. (There should be no splitting in two polynomials of degree two.)



    Assume there is a factorization
    $$
    f=(x^2+ax+b)(x^2+cx+d) ,qquad a,b,c,dinBbb F_7 .$$
    Let $une 0$ be a root of the first factor in some extension (of degree two). Then $1/u$ is also a root. We distinguish two cases.



    First case: $1/u$ is also a root of the first factor. Then $b=1$, then $d=1$, $c+a=1$, the product
    $$(x^2+ax+1)(x^2+(1-a)x+1)$$
    is reciprocal,
    and we have to look for a match of the coefficient in $x^2$, for a few values of $a$ (modulo the action $aleftrightarrow (1-a)$). There is no such $a$ with $1+a(1-a)+1=1$.



    Second case: $1/u$ is not a root of the first factor. Let $u$, $v$ be the two roots of $x^2+ax+b$. Then $1/u$, $1/v$ are the roots of the second factor, which is the reciprocal polynomial, made monic, so we expect an equality of reciprocal polynomials
    $$
    (x^2+ax+b)(1+ax+bx^2) = b(x^4+x^3+x^2+x+1) .
    $$
    The two equations obtained, $ab+a=b$ and $1+a^2+b^2=b$, have no solution in $Bbb F_7$. (The substitution of $a=b/(b+1)$ in the second equation leads to... $(b^4+b^3+b^2+b+1)/(b+1)^2=0$.)




    Note: This is arguably shorter then taking all possible polynomials $x^2+ax+b$, $bne 0$, associating $c=1-a$, $d=1/b$, and checking if the other conditions among $a,b,c,d$ are satisfied.



    Note: Computer algebra programs show that we have an irreducible polynomial in our hands. For instance using sage:



    sage: R.<x> = PolynomialRing(GF(7))
    sage: f = x^4 + x^3 + x^2 + x + 1
    sage: f.is_irreducible()
    True


    One can use then computer power also as follows to conclude. (With a slightly bigger effort than for the typing, one can also conclude humanly.) Assume $f$ splits in two (or more factors). One can check there is no root in $Bbb F_7$. Then there is a root in $Bbb F_7^2=Bbb F_49$, so the polynomials $f$ and $x^49-x$ have a common divisor. The gcd is but... we type



    sage: gcd( x^49 - x, f ) 
    1
    sage: gcd( x^(7^4) - x, f )
    x^4 + x^3 + x^2 + x + 1


    For this, one can also compute easily $(x^49-x,f)$ as a human by noting that $f$ divides $x^5-1$, so $(x^49-x,f)=(x^49-x^4+x^4-x,f)=(x^4-x,f)=(x^3-1,f)=(x^2+x+1,f)=(x^2+x+1,x+1)=(1,x+1)=1$.






    share|cite|improve this answer




























      up vote
      0
      down vote













      HINT.-If $x^4+x^3+x^2+x+1$ were reducible then we have either a linear factor with a cubic one or two quadratic factors.



      We have $$x^4+x^3+x^2+x+1=-dfrac 14(-2x^2+(sqrt5-1)x-2)(2x^2+(sqrt5+1)x+2)$$
      Since $mathbb F_7^2=1,2,4,0$ we see that $5$ is not a square modulo $7$. It follows $x^4+x^3+x^2+x+1$ is not a factor of two quadratics modulo $7$.



      Besides that a linear factor is not possible is checked easily.






      share|cite|improve this answer























      • Explain the reason of your downvote, please. If there exists a factorization in two quadratics, then it would have two distinct factorizations in two quadratics over an extension of $mathbb F_7$. This is not possible.
        – Piquito
        Aug 6 at 22:38











      • I don't know who downvoted this answer, I just upvoted it.
        – user437309
        Aug 6 at 23:05










      • @user437309: I did not say, dear friend, that it was you who put the downvote. It should be mandatory in S.E. that a downvote is always with justification exposed. That the person that put it refuses to respond shows its (mathematically speaking) ilk.
        – Piquito
        Aug 7 at 0:07










      • Hmm. Over $BbbC$ the roots of the polynomial $x^4+x^3+x^2+x+1$ are $zeta^k$ with $zeta=e^2pi i/5$, $k=1,2,3,4$. Your argument shows that the quadratic factors with real coefficients, $(x-zeta)(x-zeta^4)$ and $(x-zeta^2)(x-zeta^3)$, don't exist in $BbbF_7$ because they involve $sqrt5$. But how do you rule out a factor like $(x-zeta)(x-zeta^2)$ showing up? True, Galois theory shows that $BbbQ(sqrt5)$ is the only quadratic subfield of $BbbQ(zeta)$. That does lead to an argument, but the connection to Galois theory of finite fields runs a bit deep.
        – Jyrki Lahtonen
        Aug 7 at 5:34










      • (cont'd) In if you really want to use Galois theory here, a simpler way is to use the Frobenius automorphism $F$ of all extensions of $BbbF_7$: If $zeta$ is a root of some polynomial with coefficients in $BbbF_7$ then so is $F(zeta)=zeta^7=zeta^2$, and therefore also $F(zeta^2)=zeta^14=zeta^4$, and therefore also $F(zeta^4)=zeta^3$. That's four zeros, so any polynomial with $zeta$ as a root has degree four.
        – Jyrki Lahtonen
        Aug 7 at 5:37










      Your Answer




      StackExchange.ifUsing("editor", function ()
      return StackExchange.using("mathjaxEditing", function ()
      StackExchange.MarkdownEditor.creationCallbacks.add(function (editor, postfix)
      StackExchange.mathjaxEditing.prepareWmdForMathJax(editor, postfix, [["$", "$"], ["\\(","\\)"]]);
      );
      );
      , "mathjax-editing");

      StackExchange.ready(function()
      var channelOptions =
      tags: "".split(" "),
      id: "69"
      ;
      initTagRenderer("".split(" "), "".split(" "), channelOptions);

      StackExchange.using("externalEditor", function()
      // Have to fire editor after snippets, if snippets enabled
      if (StackExchange.settings.snippets.snippetsEnabled)
      StackExchange.using("snippets", function()
      createEditor();
      );

      else
      createEditor();

      );

      function createEditor()
      StackExchange.prepareEditor(
      heartbeatType: 'answer',
      convertImagesToLinks: true,
      noModals: false,
      showLowRepImageUploadWarning: true,
      reputationToPostImages: 10,
      bindNavPrevention: true,
      postfix: "",
      noCode: true, onDemand: true,
      discardSelector: ".discard-answer"
      ,immediatelyShowMarkdownHelp:true
      );



      );








       

      draft saved


      draft discarded


















      StackExchange.ready(
      function ()
      StackExchange.openid.initPostLogin('.new-post-login', 'https%3a%2f%2fmath.stackexchange.com%2fquestions%2f2874241%2fx4x3x2x1-irreducible-over-mathbb-f-7%23new-answer', 'question_page');

      );

      Post as a guest






























      4 Answers
      4






      active

      oldest

      votes








      4 Answers
      4






      active

      oldest

      votes









      active

      oldest

      votes






      active

      oldest

      votes








      up vote
      11
      down vote



      accepted










      Any element in any field satisfying $x^4+x^3+x^2+x+1=0$ also satisfies $x^5=1$. In other words, it has order $5$ in the multiplicative group of that field.



      The field with $p^k$ elements has a multiplicative group with $p^k-1$ elements, so it has elements of order $5$ if and only if $5$ divides $p^k-1$. This is not the case for $p=7$ and $k=1,2,3$, but it is the case for $p=7,k=4$.



      Therefore, the smallest field containing $mathbbF_7$ and roots of $x^4+x^3+x^2+x+1$ has $7^4$ elements. If that polynomial were reducible, there would be a smaller splitting field with such roots, but there isn’t.






      share|cite|improve this answer





















      • Why would there be a smaller splitting field (than that with $7^4$ elements) with the roots of $x^4+x^3+x^2+x+1$ if that polynomial were reducible?
        – user437309
        Aug 6 at 20:00










      • Take a factor of that polynomial and consider the splitting field of that factor.
        – Alon Amit
        Aug 6 at 20:01






      • 1




        I have lost count of the questions I have settled here using this argument. Welcome to the club :-)
        – Jyrki Lahtonen
        Aug 6 at 21:12










      • @JyrkiLahtonen proud to be a member!
        – Alon Amit
        Aug 6 at 22:08






      • 1




        @user437309 yes, that’s right.
        – Alon Amit
        Aug 7 at 19:45














      up vote
      11
      down vote



      accepted










      Any element in any field satisfying $x^4+x^3+x^2+x+1=0$ also satisfies $x^5=1$. In other words, it has order $5$ in the multiplicative group of that field.



      The field with $p^k$ elements has a multiplicative group with $p^k-1$ elements, so it has elements of order $5$ if and only if $5$ divides $p^k-1$. This is not the case for $p=7$ and $k=1,2,3$, but it is the case for $p=7,k=4$.



      Therefore, the smallest field containing $mathbbF_7$ and roots of $x^4+x^3+x^2+x+1$ has $7^4$ elements. If that polynomial were reducible, there would be a smaller splitting field with such roots, but there isn’t.






      share|cite|improve this answer





















      • Why would there be a smaller splitting field (than that with $7^4$ elements) with the roots of $x^4+x^3+x^2+x+1$ if that polynomial were reducible?
        – user437309
        Aug 6 at 20:00










      • Take a factor of that polynomial and consider the splitting field of that factor.
        – Alon Amit
        Aug 6 at 20:01






      • 1




        I have lost count of the questions I have settled here using this argument. Welcome to the club :-)
        – Jyrki Lahtonen
        Aug 6 at 21:12










      • @JyrkiLahtonen proud to be a member!
        – Alon Amit
        Aug 6 at 22:08






      • 1




        @user437309 yes, that’s right.
        – Alon Amit
        Aug 7 at 19:45












      up vote
      11
      down vote



      accepted







      up vote
      11
      down vote



      accepted






      Any element in any field satisfying $x^4+x^3+x^2+x+1=0$ also satisfies $x^5=1$. In other words, it has order $5$ in the multiplicative group of that field.



      The field with $p^k$ elements has a multiplicative group with $p^k-1$ elements, so it has elements of order $5$ if and only if $5$ divides $p^k-1$. This is not the case for $p=7$ and $k=1,2,3$, but it is the case for $p=7,k=4$.



      Therefore, the smallest field containing $mathbbF_7$ and roots of $x^4+x^3+x^2+x+1$ has $7^4$ elements. If that polynomial were reducible, there would be a smaller splitting field with such roots, but there isn’t.






      share|cite|improve this answer













      Any element in any field satisfying $x^4+x^3+x^2+x+1=0$ also satisfies $x^5=1$. In other words, it has order $5$ in the multiplicative group of that field.



      The field with $p^k$ elements has a multiplicative group with $p^k-1$ elements, so it has elements of order $5$ if and only if $5$ divides $p^k-1$. This is not the case for $p=7$ and $k=1,2,3$, but it is the case for $p=7,k=4$.



      Therefore, the smallest field containing $mathbbF_7$ and roots of $x^4+x^3+x^2+x+1$ has $7^4$ elements. If that polynomial were reducible, there would be a smaller splitting field with such roots, but there isn’t.







      share|cite|improve this answer













      share|cite|improve this answer



      share|cite|improve this answer











      answered Aug 6 at 19:43









      Alon Amit

      10.3k3765




      10.3k3765











      • Why would there be a smaller splitting field (than that with $7^4$ elements) with the roots of $x^4+x^3+x^2+x+1$ if that polynomial were reducible?
        – user437309
        Aug 6 at 20:00










      • Take a factor of that polynomial and consider the splitting field of that factor.
        – Alon Amit
        Aug 6 at 20:01






      • 1




        I have lost count of the questions I have settled here using this argument. Welcome to the club :-)
        – Jyrki Lahtonen
        Aug 6 at 21:12










      • @JyrkiLahtonen proud to be a member!
        – Alon Amit
        Aug 6 at 22:08






      • 1




        @user437309 yes, that’s right.
        – Alon Amit
        Aug 7 at 19:45
















      • Why would there be a smaller splitting field (than that with $7^4$ elements) with the roots of $x^4+x^3+x^2+x+1$ if that polynomial were reducible?
        – user437309
        Aug 6 at 20:00










      • Take a factor of that polynomial and consider the splitting field of that factor.
        – Alon Amit
        Aug 6 at 20:01






      • 1




        I have lost count of the questions I have settled here using this argument. Welcome to the club :-)
        – Jyrki Lahtonen
        Aug 6 at 21:12










      • @JyrkiLahtonen proud to be a member!
        – Alon Amit
        Aug 6 at 22:08






      • 1




        @user437309 yes, that’s right.
        – Alon Amit
        Aug 7 at 19:45















      Why would there be a smaller splitting field (than that with $7^4$ elements) with the roots of $x^4+x^3+x^2+x+1$ if that polynomial were reducible?
      – user437309
      Aug 6 at 20:00




      Why would there be a smaller splitting field (than that with $7^4$ elements) with the roots of $x^4+x^3+x^2+x+1$ if that polynomial were reducible?
      – user437309
      Aug 6 at 20:00












      Take a factor of that polynomial and consider the splitting field of that factor.
      – Alon Amit
      Aug 6 at 20:01




      Take a factor of that polynomial and consider the splitting field of that factor.
      – Alon Amit
      Aug 6 at 20:01




      1




      1




      I have lost count of the questions I have settled here using this argument. Welcome to the club :-)
      – Jyrki Lahtonen
      Aug 6 at 21:12




      I have lost count of the questions I have settled here using this argument. Welcome to the club :-)
      – Jyrki Lahtonen
      Aug 6 at 21:12












      @JyrkiLahtonen proud to be a member!
      – Alon Amit
      Aug 6 at 22:08




      @JyrkiLahtonen proud to be a member!
      – Alon Amit
      Aug 6 at 22:08




      1




      1




      @user437309 yes, that’s right.
      – Alon Amit
      Aug 7 at 19:45




      @user437309 yes, that’s right.
      – Alon Amit
      Aug 7 at 19:45










      up vote
      3
      down vote













      If you want to continue your approach in case you cannot come up with some clever argument like that in Alon Amit's answer, then you can do it as follows. First, note that $c=1-a$ and $d=b^-1$. Thus, the equation $ac+b+d=1$ becomes
      $$a(1-a)+b+b^-1=1,.tag*$$
      The equation $bc+ad=1$ is now
      $$b(1-a)+ab^-1=1,.$$
      Thus,
      $$a^-1big(1-b(1-a)big)=b^-1=1-b-a(1-a),.$$
      That is,
      $$1-b(1-a)=a-ab-a^2(1-a),.$$
      Consequently,
      $$(2a-1)b=-1+a-a^2+a^3=(a-1)(a^2+1),.tag#$$
      Thus, $anotin0,1,4$ for the equation above to be possible (noting that $aneq 0$ and $bneq 0$). You are left with four choices of $a$, namely, $ain2,3,5,6$.



      If $a=2$, then, using (#), we get $b=3^-1cdot 5=5cdot 5=4$, so $b^-1=2$, but then $$a(1-a)+b+b^-1=2cdot(-1)+4+2=4neq 1,.$$
      If $a=3$, then we have by (#) that $b=5^-1cdot20=-3=4$, whence
      $$a(1-a)+b+b^-1=3cdot(-2)+4+2=0neq 1,.$$
      If $a=5$, then $b=3^-1cdot (-1)=-5=2$, so $b^-1=4$, but then
      $$a(1-a)+b+b^-1=5cdot(-4)+2+4=0neq 1,.$$
      Finally, if $a=6$, then $b=4^-1cdot3=2cdot 3=-1$, whence $b^-1=-1$, and
      $$a(1-a)+b+b^-1=6cdot(-5)+(-1)+(-1)=-4neq 1,.$$
      Thus, (*) cannot be satisfied.






      share|cite|improve this answer

























        up vote
        3
        down vote













        If you want to continue your approach in case you cannot come up with some clever argument like that in Alon Amit's answer, then you can do it as follows. First, note that $c=1-a$ and $d=b^-1$. Thus, the equation $ac+b+d=1$ becomes
        $$a(1-a)+b+b^-1=1,.tag*$$
        The equation $bc+ad=1$ is now
        $$b(1-a)+ab^-1=1,.$$
        Thus,
        $$a^-1big(1-b(1-a)big)=b^-1=1-b-a(1-a),.$$
        That is,
        $$1-b(1-a)=a-ab-a^2(1-a),.$$
        Consequently,
        $$(2a-1)b=-1+a-a^2+a^3=(a-1)(a^2+1),.tag#$$
        Thus, $anotin0,1,4$ for the equation above to be possible (noting that $aneq 0$ and $bneq 0$). You are left with four choices of $a$, namely, $ain2,3,5,6$.



        If $a=2$, then, using (#), we get $b=3^-1cdot 5=5cdot 5=4$, so $b^-1=2$, but then $$a(1-a)+b+b^-1=2cdot(-1)+4+2=4neq 1,.$$
        If $a=3$, then we have by (#) that $b=5^-1cdot20=-3=4$, whence
        $$a(1-a)+b+b^-1=3cdot(-2)+4+2=0neq 1,.$$
        If $a=5$, then $b=3^-1cdot (-1)=-5=2$, so $b^-1=4$, but then
        $$a(1-a)+b+b^-1=5cdot(-4)+2+4=0neq 1,.$$
        Finally, if $a=6$, then $b=4^-1cdot3=2cdot 3=-1$, whence $b^-1=-1$, and
        $$a(1-a)+b+b^-1=6cdot(-5)+(-1)+(-1)=-4neq 1,.$$
        Thus, (*) cannot be satisfied.






        share|cite|improve this answer























          up vote
          3
          down vote










          up vote
          3
          down vote









          If you want to continue your approach in case you cannot come up with some clever argument like that in Alon Amit's answer, then you can do it as follows. First, note that $c=1-a$ and $d=b^-1$. Thus, the equation $ac+b+d=1$ becomes
          $$a(1-a)+b+b^-1=1,.tag*$$
          The equation $bc+ad=1$ is now
          $$b(1-a)+ab^-1=1,.$$
          Thus,
          $$a^-1big(1-b(1-a)big)=b^-1=1-b-a(1-a),.$$
          That is,
          $$1-b(1-a)=a-ab-a^2(1-a),.$$
          Consequently,
          $$(2a-1)b=-1+a-a^2+a^3=(a-1)(a^2+1),.tag#$$
          Thus, $anotin0,1,4$ for the equation above to be possible (noting that $aneq 0$ and $bneq 0$). You are left with four choices of $a$, namely, $ain2,3,5,6$.



          If $a=2$, then, using (#), we get $b=3^-1cdot 5=5cdot 5=4$, so $b^-1=2$, but then $$a(1-a)+b+b^-1=2cdot(-1)+4+2=4neq 1,.$$
          If $a=3$, then we have by (#) that $b=5^-1cdot20=-3=4$, whence
          $$a(1-a)+b+b^-1=3cdot(-2)+4+2=0neq 1,.$$
          If $a=5$, then $b=3^-1cdot (-1)=-5=2$, so $b^-1=4$, but then
          $$a(1-a)+b+b^-1=5cdot(-4)+2+4=0neq 1,.$$
          Finally, if $a=6$, then $b=4^-1cdot3=2cdot 3=-1$, whence $b^-1=-1$, and
          $$a(1-a)+b+b^-1=6cdot(-5)+(-1)+(-1)=-4neq 1,.$$
          Thus, (*) cannot be satisfied.






          share|cite|improve this answer













          If you want to continue your approach in case you cannot come up with some clever argument like that in Alon Amit's answer, then you can do it as follows. First, note that $c=1-a$ and $d=b^-1$. Thus, the equation $ac+b+d=1$ becomes
          $$a(1-a)+b+b^-1=1,.tag*$$
          The equation $bc+ad=1$ is now
          $$b(1-a)+ab^-1=1,.$$
          Thus,
          $$a^-1big(1-b(1-a)big)=b^-1=1-b-a(1-a),.$$
          That is,
          $$1-b(1-a)=a-ab-a^2(1-a),.$$
          Consequently,
          $$(2a-1)b=-1+a-a^2+a^3=(a-1)(a^2+1),.tag#$$
          Thus, $anotin0,1,4$ for the equation above to be possible (noting that $aneq 0$ and $bneq 0$). You are left with four choices of $a$, namely, $ain2,3,5,6$.



          If $a=2$, then, using (#), we get $b=3^-1cdot 5=5cdot 5=4$, so $b^-1=2$, but then $$a(1-a)+b+b^-1=2cdot(-1)+4+2=4neq 1,.$$
          If $a=3$, then we have by (#) that $b=5^-1cdot20=-3=4$, whence
          $$a(1-a)+b+b^-1=3cdot(-2)+4+2=0neq 1,.$$
          If $a=5$, then $b=3^-1cdot (-1)=-5=2$, so $b^-1=4$, but then
          $$a(1-a)+b+b^-1=5cdot(-4)+2+4=0neq 1,.$$
          Finally, if $a=6$, then $b=4^-1cdot3=2cdot 3=-1$, whence $b^-1=-1$, and
          $$a(1-a)+b+b^-1=6cdot(-5)+(-1)+(-1)=-4neq 1,.$$
          Thus, (*) cannot be satisfied.







          share|cite|improve this answer













          share|cite|improve this answer



          share|cite|improve this answer











          answered Aug 6 at 20:01









          Batominovski

          23.4k22779




          23.4k22779




















              up vote
              1
              down vote













              Yet an other solution. It is based on the fact that
              $$ f = x^4+x^3+x^2+x+1inBbb F_7[x] $$
              is reciprocal, and tries to use the idea in the OP. (There should be no splitting in two polynomials of degree two.)



              Assume there is a factorization
              $$
              f=(x^2+ax+b)(x^2+cx+d) ,qquad a,b,c,dinBbb F_7 .$$
              Let $une 0$ be a root of the first factor in some extension (of degree two). Then $1/u$ is also a root. We distinguish two cases.



              First case: $1/u$ is also a root of the first factor. Then $b=1$, then $d=1$, $c+a=1$, the product
              $$(x^2+ax+1)(x^2+(1-a)x+1)$$
              is reciprocal,
              and we have to look for a match of the coefficient in $x^2$, for a few values of $a$ (modulo the action $aleftrightarrow (1-a)$). There is no such $a$ with $1+a(1-a)+1=1$.



              Second case: $1/u$ is not a root of the first factor. Let $u$, $v$ be the two roots of $x^2+ax+b$. Then $1/u$, $1/v$ are the roots of the second factor, which is the reciprocal polynomial, made monic, so we expect an equality of reciprocal polynomials
              $$
              (x^2+ax+b)(1+ax+bx^2) = b(x^4+x^3+x^2+x+1) .
              $$
              The two equations obtained, $ab+a=b$ and $1+a^2+b^2=b$, have no solution in $Bbb F_7$. (The substitution of $a=b/(b+1)$ in the second equation leads to... $(b^4+b^3+b^2+b+1)/(b+1)^2=0$.)




              Note: This is arguably shorter then taking all possible polynomials $x^2+ax+b$, $bne 0$, associating $c=1-a$, $d=1/b$, and checking if the other conditions among $a,b,c,d$ are satisfied.



              Note: Computer algebra programs show that we have an irreducible polynomial in our hands. For instance using sage:



              sage: R.<x> = PolynomialRing(GF(7))
              sage: f = x^4 + x^3 + x^2 + x + 1
              sage: f.is_irreducible()
              True


              One can use then computer power also as follows to conclude. (With a slightly bigger effort than for the typing, one can also conclude humanly.) Assume $f$ splits in two (or more factors). One can check there is no root in $Bbb F_7$. Then there is a root in $Bbb F_7^2=Bbb F_49$, so the polynomials $f$ and $x^49-x$ have a common divisor. The gcd is but... we type



              sage: gcd( x^49 - x, f ) 
              1
              sage: gcd( x^(7^4) - x, f )
              x^4 + x^3 + x^2 + x + 1


              For this, one can also compute easily $(x^49-x,f)$ as a human by noting that $f$ divides $x^5-1$, so $(x^49-x,f)=(x^49-x^4+x^4-x,f)=(x^4-x,f)=(x^3-1,f)=(x^2+x+1,f)=(x^2+x+1,x+1)=(1,x+1)=1$.






              share|cite|improve this answer

























                up vote
                1
                down vote













                Yet an other solution. It is based on the fact that
                $$ f = x^4+x^3+x^2+x+1inBbb F_7[x] $$
                is reciprocal, and tries to use the idea in the OP. (There should be no splitting in two polynomials of degree two.)



                Assume there is a factorization
                $$
                f=(x^2+ax+b)(x^2+cx+d) ,qquad a,b,c,dinBbb F_7 .$$
                Let $une 0$ be a root of the first factor in some extension (of degree two). Then $1/u$ is also a root. We distinguish two cases.



                First case: $1/u$ is also a root of the first factor. Then $b=1$, then $d=1$, $c+a=1$, the product
                $$(x^2+ax+1)(x^2+(1-a)x+1)$$
                is reciprocal,
                and we have to look for a match of the coefficient in $x^2$, for a few values of $a$ (modulo the action $aleftrightarrow (1-a)$). There is no such $a$ with $1+a(1-a)+1=1$.



                Second case: $1/u$ is not a root of the first factor. Let $u$, $v$ be the two roots of $x^2+ax+b$. Then $1/u$, $1/v$ are the roots of the second factor, which is the reciprocal polynomial, made monic, so we expect an equality of reciprocal polynomials
                $$
                (x^2+ax+b)(1+ax+bx^2) = b(x^4+x^3+x^2+x+1) .
                $$
                The two equations obtained, $ab+a=b$ and $1+a^2+b^2=b$, have no solution in $Bbb F_7$. (The substitution of $a=b/(b+1)$ in the second equation leads to... $(b^4+b^3+b^2+b+1)/(b+1)^2=0$.)




                Note: This is arguably shorter then taking all possible polynomials $x^2+ax+b$, $bne 0$, associating $c=1-a$, $d=1/b$, and checking if the other conditions among $a,b,c,d$ are satisfied.



                Note: Computer algebra programs show that we have an irreducible polynomial in our hands. For instance using sage:



                sage: R.<x> = PolynomialRing(GF(7))
                sage: f = x^4 + x^3 + x^2 + x + 1
                sage: f.is_irreducible()
                True


                One can use then computer power also as follows to conclude. (With a slightly bigger effort than for the typing, one can also conclude humanly.) Assume $f$ splits in two (or more factors). One can check there is no root in $Bbb F_7$. Then there is a root in $Bbb F_7^2=Bbb F_49$, so the polynomials $f$ and $x^49-x$ have a common divisor. The gcd is but... we type



                sage: gcd( x^49 - x, f ) 
                1
                sage: gcd( x^(7^4) - x, f )
                x^4 + x^3 + x^2 + x + 1


                For this, one can also compute easily $(x^49-x,f)$ as a human by noting that $f$ divides $x^5-1$, so $(x^49-x,f)=(x^49-x^4+x^4-x,f)=(x^4-x,f)=(x^3-1,f)=(x^2+x+1,f)=(x^2+x+1,x+1)=(1,x+1)=1$.






                share|cite|improve this answer























                  up vote
                  1
                  down vote










                  up vote
                  1
                  down vote









                  Yet an other solution. It is based on the fact that
                  $$ f = x^4+x^3+x^2+x+1inBbb F_7[x] $$
                  is reciprocal, and tries to use the idea in the OP. (There should be no splitting in two polynomials of degree two.)



                  Assume there is a factorization
                  $$
                  f=(x^2+ax+b)(x^2+cx+d) ,qquad a,b,c,dinBbb F_7 .$$
                  Let $une 0$ be a root of the first factor in some extension (of degree two). Then $1/u$ is also a root. We distinguish two cases.



                  First case: $1/u$ is also a root of the first factor. Then $b=1$, then $d=1$, $c+a=1$, the product
                  $$(x^2+ax+1)(x^2+(1-a)x+1)$$
                  is reciprocal,
                  and we have to look for a match of the coefficient in $x^2$, for a few values of $a$ (modulo the action $aleftrightarrow (1-a)$). There is no such $a$ with $1+a(1-a)+1=1$.



                  Second case: $1/u$ is not a root of the first factor. Let $u$, $v$ be the two roots of $x^2+ax+b$. Then $1/u$, $1/v$ are the roots of the second factor, which is the reciprocal polynomial, made monic, so we expect an equality of reciprocal polynomials
                  $$
                  (x^2+ax+b)(1+ax+bx^2) = b(x^4+x^3+x^2+x+1) .
                  $$
                  The two equations obtained, $ab+a=b$ and $1+a^2+b^2=b$, have no solution in $Bbb F_7$. (The substitution of $a=b/(b+1)$ in the second equation leads to... $(b^4+b^3+b^2+b+1)/(b+1)^2=0$.)




                  Note: This is arguably shorter then taking all possible polynomials $x^2+ax+b$, $bne 0$, associating $c=1-a$, $d=1/b$, and checking if the other conditions among $a,b,c,d$ are satisfied.



                  Note: Computer algebra programs show that we have an irreducible polynomial in our hands. For instance using sage:



                  sage: R.<x> = PolynomialRing(GF(7))
                  sage: f = x^4 + x^3 + x^2 + x + 1
                  sage: f.is_irreducible()
                  True


                  One can use then computer power also as follows to conclude. (With a slightly bigger effort than for the typing, one can also conclude humanly.) Assume $f$ splits in two (or more factors). One can check there is no root in $Bbb F_7$. Then there is a root in $Bbb F_7^2=Bbb F_49$, so the polynomials $f$ and $x^49-x$ have a common divisor. The gcd is but... we type



                  sage: gcd( x^49 - x, f ) 
                  1
                  sage: gcd( x^(7^4) - x, f )
                  x^4 + x^3 + x^2 + x + 1


                  For this, one can also compute easily $(x^49-x,f)$ as a human by noting that $f$ divides $x^5-1$, so $(x^49-x,f)=(x^49-x^4+x^4-x,f)=(x^4-x,f)=(x^3-1,f)=(x^2+x+1,f)=(x^2+x+1,x+1)=(1,x+1)=1$.






                  share|cite|improve this answer













                  Yet an other solution. It is based on the fact that
                  $$ f = x^4+x^3+x^2+x+1inBbb F_7[x] $$
                  is reciprocal, and tries to use the idea in the OP. (There should be no splitting in two polynomials of degree two.)



                  Assume there is a factorization
                  $$
                  f=(x^2+ax+b)(x^2+cx+d) ,qquad a,b,c,dinBbb F_7 .$$
                  Let $une 0$ be a root of the first factor in some extension (of degree two). Then $1/u$ is also a root. We distinguish two cases.



                  First case: $1/u$ is also a root of the first factor. Then $b=1$, then $d=1$, $c+a=1$, the product
                  $$(x^2+ax+1)(x^2+(1-a)x+1)$$
                  is reciprocal,
                  and we have to look for a match of the coefficient in $x^2$, for a few values of $a$ (modulo the action $aleftrightarrow (1-a)$). There is no such $a$ with $1+a(1-a)+1=1$.



                  Second case: $1/u$ is not a root of the first factor. Let $u$, $v$ be the two roots of $x^2+ax+b$. Then $1/u$, $1/v$ are the roots of the second factor, which is the reciprocal polynomial, made monic, so we expect an equality of reciprocal polynomials
                  $$
                  (x^2+ax+b)(1+ax+bx^2) = b(x^4+x^3+x^2+x+1) .
                  $$
                  The two equations obtained, $ab+a=b$ and $1+a^2+b^2=b$, have no solution in $Bbb F_7$. (The substitution of $a=b/(b+1)$ in the second equation leads to... $(b^4+b^3+b^2+b+1)/(b+1)^2=0$.)




                  Note: This is arguably shorter then taking all possible polynomials $x^2+ax+b$, $bne 0$, associating $c=1-a$, $d=1/b$, and checking if the other conditions among $a,b,c,d$ are satisfied.



                  Note: Computer algebra programs show that we have an irreducible polynomial in our hands. For instance using sage:



                  sage: R.<x> = PolynomialRing(GF(7))
                  sage: f = x^4 + x^3 + x^2 + x + 1
                  sage: f.is_irreducible()
                  True


                  One can use then computer power also as follows to conclude. (With a slightly bigger effort than for the typing, one can also conclude humanly.) Assume $f$ splits in two (or more factors). One can check there is no root in $Bbb F_7$. Then there is a root in $Bbb F_7^2=Bbb F_49$, so the polynomials $f$ and $x^49-x$ have a common divisor. The gcd is but... we type



                  sage: gcd( x^49 - x, f ) 
                  1
                  sage: gcd( x^(7^4) - x, f )
                  x^4 + x^3 + x^2 + x + 1


                  For this, one can also compute easily $(x^49-x,f)$ as a human by noting that $f$ divides $x^5-1$, so $(x^49-x,f)=(x^49-x^4+x^4-x,f)=(x^4-x,f)=(x^3-1,f)=(x^2+x+1,f)=(x^2+x+1,x+1)=(1,x+1)=1$.







                  share|cite|improve this answer













                  share|cite|improve this answer



                  share|cite|improve this answer











                  answered Aug 6 at 21:32









                  dan_fulea

                  4,2171211




                  4,2171211




















                      up vote
                      0
                      down vote













                      HINT.-If $x^4+x^3+x^2+x+1$ were reducible then we have either a linear factor with a cubic one or two quadratic factors.



                      We have $$x^4+x^3+x^2+x+1=-dfrac 14(-2x^2+(sqrt5-1)x-2)(2x^2+(sqrt5+1)x+2)$$
                      Since $mathbb F_7^2=1,2,4,0$ we see that $5$ is not a square modulo $7$. It follows $x^4+x^3+x^2+x+1$ is not a factor of two quadratics modulo $7$.



                      Besides that a linear factor is not possible is checked easily.






                      share|cite|improve this answer























                      • Explain the reason of your downvote, please. If there exists a factorization in two quadratics, then it would have two distinct factorizations in two quadratics over an extension of $mathbb F_7$. This is not possible.
                        – Piquito
                        Aug 6 at 22:38











                      • I don't know who downvoted this answer, I just upvoted it.
                        – user437309
                        Aug 6 at 23:05










                      • @user437309: I did not say, dear friend, that it was you who put the downvote. It should be mandatory in S.E. that a downvote is always with justification exposed. That the person that put it refuses to respond shows its (mathematically speaking) ilk.
                        – Piquito
                        Aug 7 at 0:07










                      • Hmm. Over $BbbC$ the roots of the polynomial $x^4+x^3+x^2+x+1$ are $zeta^k$ with $zeta=e^2pi i/5$, $k=1,2,3,4$. Your argument shows that the quadratic factors with real coefficients, $(x-zeta)(x-zeta^4)$ and $(x-zeta^2)(x-zeta^3)$, don't exist in $BbbF_7$ because they involve $sqrt5$. But how do you rule out a factor like $(x-zeta)(x-zeta^2)$ showing up? True, Galois theory shows that $BbbQ(sqrt5)$ is the only quadratic subfield of $BbbQ(zeta)$. That does lead to an argument, but the connection to Galois theory of finite fields runs a bit deep.
                        – Jyrki Lahtonen
                        Aug 7 at 5:34










                      • (cont'd) In if you really want to use Galois theory here, a simpler way is to use the Frobenius automorphism $F$ of all extensions of $BbbF_7$: If $zeta$ is a root of some polynomial with coefficients in $BbbF_7$ then so is $F(zeta)=zeta^7=zeta^2$, and therefore also $F(zeta^2)=zeta^14=zeta^4$, and therefore also $F(zeta^4)=zeta^3$. That's four zeros, so any polynomial with $zeta$ as a root has degree four.
                        – Jyrki Lahtonen
                        Aug 7 at 5:37














                      up vote
                      0
                      down vote













                      HINT.-If $x^4+x^3+x^2+x+1$ were reducible then we have either a linear factor with a cubic one or two quadratic factors.



                      We have $$x^4+x^3+x^2+x+1=-dfrac 14(-2x^2+(sqrt5-1)x-2)(2x^2+(sqrt5+1)x+2)$$
                      Since $mathbb F_7^2=1,2,4,0$ we see that $5$ is not a square modulo $7$. It follows $x^4+x^3+x^2+x+1$ is not a factor of two quadratics modulo $7$.



                      Besides that a linear factor is not possible is checked easily.






                      share|cite|improve this answer























                      • Explain the reason of your downvote, please. If there exists a factorization in two quadratics, then it would have two distinct factorizations in two quadratics over an extension of $mathbb F_7$. This is not possible.
                        – Piquito
                        Aug 6 at 22:38











                      • I don't know who downvoted this answer, I just upvoted it.
                        – user437309
                        Aug 6 at 23:05










                      • @user437309: I did not say, dear friend, that it was you who put the downvote. It should be mandatory in S.E. that a downvote is always with justification exposed. That the person that put it refuses to respond shows its (mathematically speaking) ilk.
                        – Piquito
                        Aug 7 at 0:07










                      • Hmm. Over $BbbC$ the roots of the polynomial $x^4+x^3+x^2+x+1$ are $zeta^k$ with $zeta=e^2pi i/5$, $k=1,2,3,4$. Your argument shows that the quadratic factors with real coefficients, $(x-zeta)(x-zeta^4)$ and $(x-zeta^2)(x-zeta^3)$, don't exist in $BbbF_7$ because they involve $sqrt5$. But how do you rule out a factor like $(x-zeta)(x-zeta^2)$ showing up? True, Galois theory shows that $BbbQ(sqrt5)$ is the only quadratic subfield of $BbbQ(zeta)$. That does lead to an argument, but the connection to Galois theory of finite fields runs a bit deep.
                        – Jyrki Lahtonen
                        Aug 7 at 5:34










                      • (cont'd) In if you really want to use Galois theory here, a simpler way is to use the Frobenius automorphism $F$ of all extensions of $BbbF_7$: If $zeta$ is a root of some polynomial with coefficients in $BbbF_7$ then so is $F(zeta)=zeta^7=zeta^2$, and therefore also $F(zeta^2)=zeta^14=zeta^4$, and therefore also $F(zeta^4)=zeta^3$. That's four zeros, so any polynomial with $zeta$ as a root has degree four.
                        – Jyrki Lahtonen
                        Aug 7 at 5:37












                      up vote
                      0
                      down vote










                      up vote
                      0
                      down vote









                      HINT.-If $x^4+x^3+x^2+x+1$ were reducible then we have either a linear factor with a cubic one or two quadratic factors.



                      We have $$x^4+x^3+x^2+x+1=-dfrac 14(-2x^2+(sqrt5-1)x-2)(2x^2+(sqrt5+1)x+2)$$
                      Since $mathbb F_7^2=1,2,4,0$ we see that $5$ is not a square modulo $7$. It follows $x^4+x^3+x^2+x+1$ is not a factor of two quadratics modulo $7$.



                      Besides that a linear factor is not possible is checked easily.






                      share|cite|improve this answer















                      HINT.-If $x^4+x^3+x^2+x+1$ were reducible then we have either a linear factor with a cubic one or two quadratic factors.



                      We have $$x^4+x^3+x^2+x+1=-dfrac 14(-2x^2+(sqrt5-1)x-2)(2x^2+(sqrt5+1)x+2)$$
                      Since $mathbb F_7^2=1,2,4,0$ we see that $5$ is not a square modulo $7$. It follows $x^4+x^3+x^2+x+1$ is not a factor of two quadratics modulo $7$.



                      Besides that a linear factor is not possible is checked easily.







                      share|cite|improve this answer















                      share|cite|improve this answer



                      share|cite|improve this answer








                      edited Aug 7 at 1:16


























                      answered Aug 6 at 21:33









                      Piquito

                      17.4k31234




                      17.4k31234











                      • Explain the reason of your downvote, please. If there exists a factorization in two quadratics, then it would have two distinct factorizations in two quadratics over an extension of $mathbb F_7$. This is not possible.
                        – Piquito
                        Aug 6 at 22:38











                      • I don't know who downvoted this answer, I just upvoted it.
                        – user437309
                        Aug 6 at 23:05










                      • @user437309: I did not say, dear friend, that it was you who put the downvote. It should be mandatory in S.E. that a downvote is always with justification exposed. That the person that put it refuses to respond shows its (mathematically speaking) ilk.
                        – Piquito
                        Aug 7 at 0:07










                      • Hmm. Over $BbbC$ the roots of the polynomial $x^4+x^3+x^2+x+1$ are $zeta^k$ with $zeta=e^2pi i/5$, $k=1,2,3,4$. Your argument shows that the quadratic factors with real coefficients, $(x-zeta)(x-zeta^4)$ and $(x-zeta^2)(x-zeta^3)$, don't exist in $BbbF_7$ because they involve $sqrt5$. But how do you rule out a factor like $(x-zeta)(x-zeta^2)$ showing up? True, Galois theory shows that $BbbQ(sqrt5)$ is the only quadratic subfield of $BbbQ(zeta)$. That does lead to an argument, but the connection to Galois theory of finite fields runs a bit deep.
                        – Jyrki Lahtonen
                        Aug 7 at 5:34










                      • (cont'd) In if you really want to use Galois theory here, a simpler way is to use the Frobenius automorphism $F$ of all extensions of $BbbF_7$: If $zeta$ is a root of some polynomial with coefficients in $BbbF_7$ then so is $F(zeta)=zeta^7=zeta^2$, and therefore also $F(zeta^2)=zeta^14=zeta^4$, and therefore also $F(zeta^4)=zeta^3$. That's four zeros, so any polynomial with $zeta$ as a root has degree four.
                        – Jyrki Lahtonen
                        Aug 7 at 5:37
















                      • Explain the reason of your downvote, please. If there exists a factorization in two quadratics, then it would have two distinct factorizations in two quadratics over an extension of $mathbb F_7$. This is not possible.
                        – Piquito
                        Aug 6 at 22:38











                      • I don't know who downvoted this answer, I just upvoted it.
                        – user437309
                        Aug 6 at 23:05










                      • @user437309: I did not say, dear friend, that it was you who put the downvote. It should be mandatory in S.E. that a downvote is always with justification exposed. That the person that put it refuses to respond shows its (mathematically speaking) ilk.
                        – Piquito
                        Aug 7 at 0:07










                      • Hmm. Over $BbbC$ the roots of the polynomial $x^4+x^3+x^2+x+1$ are $zeta^k$ with $zeta=e^2pi i/5$, $k=1,2,3,4$. Your argument shows that the quadratic factors with real coefficients, $(x-zeta)(x-zeta^4)$ and $(x-zeta^2)(x-zeta^3)$, don't exist in $BbbF_7$ because they involve $sqrt5$. But how do you rule out a factor like $(x-zeta)(x-zeta^2)$ showing up? True, Galois theory shows that $BbbQ(sqrt5)$ is the only quadratic subfield of $BbbQ(zeta)$. That does lead to an argument, but the connection to Galois theory of finite fields runs a bit deep.
                        – Jyrki Lahtonen
                        Aug 7 at 5:34










                      • (cont'd) In if you really want to use Galois theory here, a simpler way is to use the Frobenius automorphism $F$ of all extensions of $BbbF_7$: If $zeta$ is a root of some polynomial with coefficients in $BbbF_7$ then so is $F(zeta)=zeta^7=zeta^2$, and therefore also $F(zeta^2)=zeta^14=zeta^4$, and therefore also $F(zeta^4)=zeta^3$. That's four zeros, so any polynomial with $zeta$ as a root has degree four.
                        – Jyrki Lahtonen
                        Aug 7 at 5:37















                      Explain the reason of your downvote, please. If there exists a factorization in two quadratics, then it would have two distinct factorizations in two quadratics over an extension of $mathbb F_7$. This is not possible.
                      – Piquito
                      Aug 6 at 22:38





                      Explain the reason of your downvote, please. If there exists a factorization in two quadratics, then it would have two distinct factorizations in two quadratics over an extension of $mathbb F_7$. This is not possible.
                      – Piquito
                      Aug 6 at 22:38













                      I don't know who downvoted this answer, I just upvoted it.
                      – user437309
                      Aug 6 at 23:05




                      I don't know who downvoted this answer, I just upvoted it.
                      – user437309
                      Aug 6 at 23:05












                      @user437309: I did not say, dear friend, that it was you who put the downvote. It should be mandatory in S.E. that a downvote is always with justification exposed. That the person that put it refuses to respond shows its (mathematically speaking) ilk.
                      – Piquito
                      Aug 7 at 0:07




                      @user437309: I did not say, dear friend, that it was you who put the downvote. It should be mandatory in S.E. that a downvote is always with justification exposed. That the person that put it refuses to respond shows its (mathematically speaking) ilk.
                      – Piquito
                      Aug 7 at 0:07












                      Hmm. Over $BbbC$ the roots of the polynomial $x^4+x^3+x^2+x+1$ are $zeta^k$ with $zeta=e^2pi i/5$, $k=1,2,3,4$. Your argument shows that the quadratic factors with real coefficients, $(x-zeta)(x-zeta^4)$ and $(x-zeta^2)(x-zeta^3)$, don't exist in $BbbF_7$ because they involve $sqrt5$. But how do you rule out a factor like $(x-zeta)(x-zeta^2)$ showing up? True, Galois theory shows that $BbbQ(sqrt5)$ is the only quadratic subfield of $BbbQ(zeta)$. That does lead to an argument, but the connection to Galois theory of finite fields runs a bit deep.
                      – Jyrki Lahtonen
                      Aug 7 at 5:34




                      Hmm. Over $BbbC$ the roots of the polynomial $x^4+x^3+x^2+x+1$ are $zeta^k$ with $zeta=e^2pi i/5$, $k=1,2,3,4$. Your argument shows that the quadratic factors with real coefficients, $(x-zeta)(x-zeta^4)$ and $(x-zeta^2)(x-zeta^3)$, don't exist in $BbbF_7$ because they involve $sqrt5$. But how do you rule out a factor like $(x-zeta)(x-zeta^2)$ showing up? True, Galois theory shows that $BbbQ(sqrt5)$ is the only quadratic subfield of $BbbQ(zeta)$. That does lead to an argument, but the connection to Galois theory of finite fields runs a bit deep.
                      – Jyrki Lahtonen
                      Aug 7 at 5:34












                      (cont'd) In if you really want to use Galois theory here, a simpler way is to use the Frobenius automorphism $F$ of all extensions of $BbbF_7$: If $zeta$ is a root of some polynomial with coefficients in $BbbF_7$ then so is $F(zeta)=zeta^7=zeta^2$, and therefore also $F(zeta^2)=zeta^14=zeta^4$, and therefore also $F(zeta^4)=zeta^3$. That's four zeros, so any polynomial with $zeta$ as a root has degree four.
                      – Jyrki Lahtonen
                      Aug 7 at 5:37




                      (cont'd) In if you really want to use Galois theory here, a simpler way is to use the Frobenius automorphism $F$ of all extensions of $BbbF_7$: If $zeta$ is a root of some polynomial with coefficients in $BbbF_7$ then so is $F(zeta)=zeta^7=zeta^2$, and therefore also $F(zeta^2)=zeta^14=zeta^4$, and therefore also $F(zeta^4)=zeta^3$. That's four zeros, so any polynomial with $zeta$ as a root has degree four.
                      – Jyrki Lahtonen
                      Aug 7 at 5:37












                       

                      draft saved


                      draft discarded


























                       


                      draft saved


                      draft discarded














                      StackExchange.ready(
                      function ()
                      StackExchange.openid.initPostLogin('.new-post-login', 'https%3a%2f%2fmath.stackexchange.com%2fquestions%2f2874241%2fx4x3x2x1-irreducible-over-mathbb-f-7%23new-answer', 'question_page');

                      );

                      Post as a guest













































































                      Comments

                      Popular posts from this blog

                      Color the edges and diagonals of a regular polygon

                      Relationship between determinant of matrix and determinant of adjoint?

                      What is the equation of a 3D cone with generalised tilt?